Загрузка страницы

The WKB Approximation for Energy less than V - Finding the Transmission coefficient

In this video I will explain the semiclassical (WKB, WKBJ) approximation for the case when the Energy is less than the potential. We will find a formula for the transmission coefficient.

If you enjoy my content, please consider checking out my Patreon!
www.patreon.com/nickheumann

Also, consider subscribing and following me on my socials!

twitter: twitter.com/nickheumann
Instagram: instagram.com/nicolasheumann
twitch: www.twitch.tv/nickheumann

My name is Nick Heumann, I am a recently graduated physicist. I love to teach physics, so I decided to give YouTube a try. English is not my first language, but I hope that you can understand me well enough regardless.
▬ Contents of this video ▬▬▬▬▬▬▬▬▬▬
00:00 Explaining the Situation
02:00 What does the Wavefunction look like in the region of the potential?
02:50 Changing our previous result for this new case
05:27 Determining the Transmission coefficient
08:20 Support my patreon!

Видео The WKB Approximation for Energy less than V - Finding the Transmission coefficient канала Nick Heumann
Показать
Комментарии отсутствуют
Введите заголовок:

Введите адрес ссылки:

Введите адрес видео с YouTube:

Зарегистрируйтесь или войдите с
Информация о видео
4 июня 2022 г. 5:20:32
00:08:37
Другие видео канала
Griffiths Problem 4.4 (3rd ed) Solution: The OTHER Solution to the Angular EquationGriffiths Problem 4.4 (3rd ed) Solution: The OTHER Solution to the Angular EquationGriffiths QM Problem 4.8: Infinite Spherical Well for l=1Griffiths QM Problem 4.8: Infinite Spherical Well for l=1Proving that the solutions to the Schrödinger Equation can be separated into even and odd solutionsProving that the solutions to the Schrödinger Equation can be separated into even and odd solutionsGriffiths Intro to QM - Problem 8.15: The ULTIMATE WKB Problem -HARDEST ONE YETGriffiths Intro to QM - Problem 8.15: The ULTIMATE WKB Problem -HARDEST ONE YETProving orthogonality and Completeness of Infinite Square Well WavefunctionsProving orthogonality and Completeness of Infinite Square Well WavefunctionsThe Integral You HAVE TO KNOW to MASTER Quantum Mechanics!The Integral You HAVE TO KNOW to MASTER Quantum Mechanics!How do LASERs work? A Quantum Mechanical Explanation Using Perturbation TheoryHow do LASERs work? A Quantum Mechanical Explanation Using Perturbation TheoryHow to determine the 4 Wavefunctions of the Hydrogen Atom for n=2How to determine the 4 Wavefunctions of the Hydrogen Atom for n=2How To Go From Arcsin(x) to Ln (natural logarithm) (Arcsin - Log Formulation)How To Go From Arcsin(x) to Ln (natural logarithm) (Arcsin - Log Formulation)Griffiths QM 6.26 (3rd ed) 6.28 (2nd ed): Hyperfine Splitting for Muonic H, Positronium, MuoniumGriffiths QM 6.26 (3rd ed) 6.28 (2nd ed): Hyperfine Splitting for Muonic H, Positronium, MuoniumGriffiths QM 2.27 Solution: Finding Transmission coefficient for double delta potentialGriffiths QM 2.27 Solution: Finding Transmission coefficient for double delta potentialWhy we need QFT & Derivation of Klein-Gordon Langriangian DensityWhy we need QFT & Derivation of Klein-Gordon Langriangian DensityHow to Solve The Infinite Spherical Well (FULLY EXPLAINED)How to Solve The Infinite Spherical Well (FULLY EXPLAINED)Griffiths QM 6.13 (3d edition) Solution: Relativistic Correction for Quantum Harmonic OscillatorGriffiths QM 6.13 (3d edition) Solution: Relativistic Correction for Quantum Harmonic OscillatorWKB Approximation Connection Formulas Example: Potential Well with no Vertical WallsWKB Approximation Connection Formulas Example: Potential Well with no Vertical WallsVariational Principle in QM Example 1: Quantum Harmonic OscillatorVariational Principle in QM Example 1: Quantum Harmonic OscillatorGriffiths QM 4.15 Solution: Proving Radial Wavefunction and finding expectation values for l=n-1Griffiths QM 4.15 Solution: Proving Radial Wavefunction and finding expectation values for l=n-1Solving the Schrödinger Equation for the Infinite Potential Well (Particle in a square box)Solving the Schrödinger Equation for the Infinite Potential Well (Particle in a square box)Griffiths QM Problem 2.37 (3rd edition): Find expectation values for nth stationary state of QHOGriffiths QM Problem 2.37 (3rd edition): Find expectation values for nth stationary state of QHOGriffiths Intro to QM 7.11 (3rd edition) solution: ground state energy of H using Ae^(-br^2)Griffiths Intro to QM 7.11 (3rd edition) solution: ground state energy of H using Ae^(-br^2)
Яндекс.Метрика